OB Questions

Pataasin ang iyong marka sa homework at exams ngayon gamit ang Quizwiz!

The nurse is preparing to assist in administering neonatal resuscitation with a ventilation bag and mask because the newborn is apneic, gasping, and has a heart rate below 100 beats/min. The nurse should understand that how many ventilations per minute should be delivered to this neonate? 1.20 to 40 breaths/min 2.40 to 60 breaths/min 3.70 to 80 breaths/min 4.80 to 100 breaths/min

2.40 to 60 breaths/min

The nurse is preparing to care for a newborn receiving phototherapy. Which interventions should be included in the plan of care? Select all that apply. 1.Avoid stimulation. 2.Decrease fluid intake. 3.Expose all of the newborn's skin. 4.Monitor skin temperature closely. 5.Reposition the newborn every 2 hours. 6.Cover the newborn's eyes with eye shields or patches.

4.Monitor skin temperature closely. 5.Reposition the newborn every 2 hours. 6.Cover the newborn's eyes with eye shields or patches.

Butorphanol tartrate (Stadol) is prescribed for a woman in labor, and the woman asks the nurse about the purpose of the medication. The nurse should make which most appropriate response? 1."The medication provides pain relief during labor." 2."The medication will help prevent any nausea and vomiting." 3."The medication will assist in increasing the contractions." 4."The medication prevents respiratory depression in the newborn infant."

1."The medication provides pain relief during labor."

The nurse is about the give a Type 2 diabetic her insulin before breakfast on her first day postpartum. Which of the following answers best describes insulin requirements immediately postpartum? 1. Lower than during her pregnancy 2. Higher than during her pregnancy 3. Lower than before she became pregnant 4. Higher than before she became pregnant

3. PP insulin requirements are usually significantly lower than prepregnancy requirements. Occasionally, clients may require little to no insulin during the first 24 to 48 hours postpartum.

Which measure would be least effective in preventing postpartum hemorrhage? 1.Administer Methergine 0.2 mg every 6 hours for 4 doses as ordered 2.Encourage the woman to void every 2 hours 3.Massage the fundus every hour for the first 24 hours following birth 4.Teach the woman the importance of rest and nutrition to enhance healing

3. The fundus should be massaged only when boggy or soft. Massaging a firm fundus could cause it to relax. Responses 1, 2, and 4 are all effective measures to enhance and maintain contraction of the uterus and to facilitate healing.

When caring for a client who's having her second baby, the nurse can anticipate the client's labor will be which of the following? 1. Shorter than her first labor 2. About half as long as her first labor 3. About the same length of time as her first labor 4. A length of time that can't be determined based on her first labor

Answer: 2 RATIONALES: A woman having her second baby can anticipate a labor about half as long as her first labor. The other options are incorrect.

Several minutes after a vaginal delivery, nursing assessment reveals blood gushing from the client's vagina, umbilical cord lengthening, and a globular-shaped uterus. The nurse should suspect which condition? 1. Cervical or vaginal laceration 2. Placental separation 3. Postpartum hemorrhage 4. Uterine involution

Answer: 2 RATIONALES: Placental separation is characterized by a sudden gush or trickle of blood from the vagina, further protrusion of the umbilical cord from the vagina, a globular-shaped uterus, and an increase in fundal height. With cervical or vaginal laceration, the nurse notes a consistent flow of bright red blood from the vagina. With postpartum hemorrhage, usually caused by uterine atony, the uterus isn't globular. Uterine involution can't begin until the placenta has been delivered.

The postpartum nurse teaches a mother how to give a bath to the newborn infant and observes the mother performing the procedure. Which observation indicates a lack of understanding of the instructions? 1. The mother bathes the newborn infant after a feeding. 2. The mother states that she would gather all supplies before the bath is started. 3. The mother states that she would never leave the newborn infant in the tub of water alone. 4. The mother fills a clean basin or sink with 2 to 3 inches of water and then checks the temperature with her wrist.

1. The mother bathes the newborn infant after a feeding. It is not advisable to bathe a newborn infant after a feeding because handling may cause regurgitation.

During the 3rd PP day, which of the following observations about the client would the nurse be most likely to make? 1. The client appears interested in learning about neonatal care 2. The client talks a lot about her birth experience 3. The client sleeps whenever the neonate isn't present 4. The client requests help in choosing a name for the neonate.

1. The third to tenth days of PP care are the "taking-hold" phase, in which the new mother strives for independence and is eager for her neonate. The other options describe the phase in which the mother relives her birth experience.

Methergine or pitocin are prescribed for a client with PP hemorrhage. Before administering the medication(s), the nurse contacts the health provider who prescribed the medication(s) in which of the following conditions is documented in the client's medical history? 1. Peripheral vascular disease 2. Hypothyroidism 3. Hypotension 4. Type 1 diabetes

1. These medications are avoided in clients with significant cardiovascular disease, peripheral disease, hypertension, eclampsia, or preeclampsia. These conditions are worsened by the vasoconstriction effects of these medications.

A PP nurse is providing instructions to a woman after delivery of a healthy newborn infant. The nurse instructs the mother that she should expect normal bowel elimination to return: 1. On the day of the delivery 2. 3 days PP 3. 7 days PP 4. within 2 weeks PP

2. After birth, the nurse should auscultate the woman's abdomen in all four quadrants to determine the return of bowel sounds. Normal bowel elimination usually returns 2 to 3 days PP. Surgery, anesthesia, and the use of narcotics and pain control agents also contribute to the longer period of altered bowel function.

A PP nurse is assessing a mother who delivered a healthy newborn infant by C-section. The nurse is assessing for signs and symptoms of superficial venous thrombosis. Which of the following signs or symptoms would the nurse note if superficial venous thrombosis were present? 1. Paleness of the calf area 2. Enlarged, hardened veins 3. Coolness of the calf area 4. Palpable dorsalis pedis pulses

2. Thrombosis of the superficial veins is usually accompanied by signs and symptoms of inflammation. These include swelling of the involved extremity and redness, tenderness, and warmth.

A client who is positive for human immunodeficiency virus (HIV) delivers a newborn infant. The nurse provides instructions to help the client regarding care of her infant. Which client statement indicates the need for further instruction? 1."I will be sure to wash my hands before and after bathroom use." 2."I need to breast-feed, especially for the first 6 weeks postpartum." 3."Support groups are available to assist me with understanding my diagnosis of HIV." 4."My newborn infant should be on antiviral medications for the first 6 weeks after delivery."

2."I need to breast-feed, especially for the first 6 weeks postpartum."

The nurse is preparing to check the respirations of a newborn who was just delivered. The nurse performs the procedure and should determine that the respiratory rate is normal if which respiratory rate is noted? 1.A respiratory rate of 20 breaths/min 2.A respiratory rate of 40 breaths/min 3.A respiratory rate of 70 breaths/min 4.A respiratory rate of 80 breaths/min

2.A respiratory rate of 40 breaths/min

A nurse is caring for a PP woman who has received epidural anesthesia and is monitoring the woman for the presence of a vulva hematoma. Which of the following assessment findings would best indicate the presence of a hematoma? 1.Complaints of a tearing sensation 2.Complaints of intense pain 3.Changes in vital signs 4.Signs of heavy bruising

3. Because the woman has had epidural anesthesia and is anesthetized, she cannot feel pain, pressure, or a tearing sensation. Changes in vitals indicate hypovolemia in the anesthetized PP woman with vulvar hematoma. Heavy bruising may be visualized, but vital sign changes indicate hematoma caused by blood collection in the perineal tissues.

A nurse is preparing to perform a fundal assessment on a postpartum client. The initial nursing action in performing this assessment is which of the following? 1. Ask the client to turn on her side 2. Ask the client to lie flat on her back with the knees and legs flat and straight. 3. Ask the mother to urinate and empty her bladder 4. Massage the fundus gently before determining the level of the fundus.

3. Before starting the fundal assessment, the nurse should ask the mother to empty her bladder so that an accurate assessment can be done. When the nurse is performing fundal assessment, the nurse asks the woman to lie flat on her back with the knees flexed. Massaging the fundus is not appropriate unless the fundus is boggy and soft, and then it should be massaged gently until firm.

A client is complaining of painful contractions, or afterpains, on postpartum day 2. Which of the following conditions could increase the severity of afterpains? 1. Bottle-feeding 2. Diabetes 3. Multiple gestation 4. Primiparity

3. Multiple gestation, breastfeeding, multiparity, and conditions that cause overdistention of the uterus will increase the intensity of after-pains. Bottle-feeding and diabetes aren't directly associated with increasing severity of afterpains unless the client has delivered a macrosomic infant.

The nurse is developing a plan of care for a preterm newborn infant. The nurse develops measures to provide skin care, knowing that the preterm newborn infant's skin appears in what way? 1.Thin and gelatinous, with increased subcutaneous fat 2.Thin and gelatinous, with increased amounts of brown fat 3. Reddened, translucent, and gelatinous, with decreased amounts of subcutaneous fat 4. With fine downy hair on thin epidermal and dermal layers, with increased amount of brown fat

3. Reddened, translucent, and gelatinous, with decreased amounts of subcutaneous fat

Which of the following interventions would be helpful to a breastfeeding mother who is experiencing engorged breasts? 1. Applying ice 2. Applying a breast binder 3. Teaching how to express her breasts in a warm shower 4. Administering bromocriptine (Parlodel)

3. Teaching the client how to express her breasts in a warm shower aids with let-down and will give temporary relief. Ice can promote comfort by vasoconstriction, numbing, and discouraging further letdown of milk.

A new mother received epidural anesthesia during labor and had a forceps delivery after pushing 2 hours. At 6 hours PP, her systolic blood pressure has dropped 20 points, her diastolic BP has dropped 10 points, and her pulse is 120 beats per minute. The client is anxious and restless. On further assessment, a vulvar hematoma is verified. After notifying the health care provider, the nurse immediately plans to: 1. Monitor fundal height 2. Apply perineal pressure 3. Prepare the client for surgery. 4. Reassure the client

3. The use of an epidural, prolonged second stage labor and forceps delivery are predisposing factors for hematoma formation, and a collection of up to 500 ml of blood can occur in the vaginal area. Although the other options may be implemented, the immediate action would be to prepare the client for surgery to stop the bleeding.

Which of the following circumstances is most likely to cause uterine atony and lead to PP hemorrhage? 1. Hypertension 2. Cervical and vaginal tears 3. Urine retention 4. Endometritis

3. Urine retention causes a distended bladder to displace the uterus above the umbilicus and to the side, which prevents the uterus from contracting. The uterus needs to remain contracted if bleeding is to stay within normal limits. Cervical and vaginal tears can cause PP hemorrhage but are less common occurrences in the PP period.

A nurse is providing instructions to a mother who has been diagnosed with mastitis. Which of the following statements if made by the mother indicates a need for further teaching? 1. "I need to take antibiotics, and I should begin to feel better in 24-48 hours." 2. "I can use analgesics to assist in alleviating some of the discomfort." 3. "I need to wear a supportive bra to relieve the discomfort." 4. "I need to stop breastfeeding until this condition resolves."

4. In most cases, the mother can continue to breast feed with both breasts. If the affected breast is too sore, the mother can pump the breast gently. Regular emptying of the breast is important to prevent abscess formation. Antibiotic therapy assists in resolving the mastitis within 24-48 hours. Additional supportive measures include ice packs, breast supports, and analgesics.

Which of the following complications is most likely responsible for a delayed postpartum hemorrhage? 1. Cervical laceration 2. Clotting deficiency 3. Perineal laceration 4. Uterine subinvolution

4. Late postpartum bleeding is often the result of subinvolution of the uterus. Retained products of conception or infection often cause subinvolution. Cervical or perineal lacerations can cause an immediate postpartum hemorrhage. A client with a clotting deficiency may also have an immediate PP hemorrhage if the deficiency isn't corrected at the time of delivery.

Which type of lochia should the nurse expect to find in a client 2 days PP? 1. Foul-smelling 2. Lochia serosa 3. Lochia alba 4. Lochia rubra

4. Lochia rubra. Lochia rubra lasts 3 to 4 days PP; Lochia serosa begins 3 days PP and lasts 10 days; Lochia alba begins 2 wks PP and lasts up to 6 weeks. Foul-smelling lochia is an abnormal finding and indicates an infection.

Perineal care is an important infection control measure. When evaluating a postpartum woman's perineal care technique, the nurse would recognize the need for further instruction if the woman: 1.Uses soap and warm water to wash the vulva and perineum 2.Washes from symphysis pubis back to episiotomy 3.Changes her perineal pad every 2 - 3 hours 4.Uses the peribottle to rinse upward into her vagina

4. Responses 1, 2, and 3 are all appropriate measures. The peribottle should be used in a backward direction over the perineum. The flow should never be directed upward into the vagina since debris would be forced upward into the uterus through the still-open cervix.

Before giving a PP client the rubella vaccine, which of the following facts should the nurse include in client teaching? 1. The vaccine is safe in clients with egg allergies 2. Breast-feeding isn't compatible with the vaccine 3. Transient arthralgia and rash are common adverse effects 4. The client should avoid getting pregnant for 3 months after the vaccine because the vaccine has teratogenic effects

4. The client must understand that she must not become pregnant for 3 months after the vaccination because of its potential teratogenic effects. The rubella vaccine is made from duck eggs so an allergic reaction may occur in clients with egg allergies. The virus is not transmitted into the breast milk, so clients may continue to breastfeed after the vaccination. Transient arthralgia and rash are common adverse effects of the vaccine.

A postpartum nurse is taking the vital signs of a woman who delivered a healthy newborn infant 4 hours ago. The nurse notes that the mother's temperature is 100.2*F. Which of the following actions would be most appropriate? 1. Retake the temperature in 15 minutes 2. Notify the physician 3. Document the findings 4. Increase hydration by encouraging oral fluids

4. The most appropriate action is to increase hydration by encouraging oral fluids, which should bring the temperature to a normal reading. Although the nurse would document the findings, the most appropriate action would be to increase the hydration. The mother's temperature may be taken every 4 hours while she is awake. Temperatures up to 100.4 (38 C) in the first 24 hours after birth are often related to the dehydrating effects of labor.

What type of milk is present in the breasts 7 to 10 days PP? 1. Colostrum 2. Hind milk 3. Mature milk 4. Transitional milk

4. Transitional milk comes after colostrum and usually lasts until 2 weeks PP

The mother of a 1-month-old infant is bottle-feeding her infant and asks the nurse about the stomach capacity of an infant. What should the nurse tell the client is the stomach capacity of a 1-month-old infant? 1.10 to 20 mL 2.30 to 90 mL 3.75 to 100 mL 4.90 to 150 mL

4.90 to 150 mL

The home care nurse is visiting a mother 1 week after she gave birth to an infant who is at risk for developing neonatal congenital syphilis. After teaching the mother about the signs and symptoms of this disorder, the nurse instructs the mother to monitor the infant for which finding? 1.Loose stools 2.High-pitched cry 3.Vigorous feeding habits 4.A copper-colored skin rash

4.A copper-colored skin rash

The nurse in the labor room measures the Apgar score in a newborn infant and notes that the score is 4. Which action by the nurse has highest priority? 1.Initiate an intravenous (IV) line on the newborn infant. 2.Place the newborn infant on a cardiorespiratory monitor. 3.Place the newborn infant in the radiant warmer incubator. 4.Administer oxygen via resuscitation bag to the newborn infant.

4.Administer oxygen via resuscitation bag to the newborn infant.

The nurse is assessing a newborn infant with a diagnosis of hiatal hernia. Which findings would the nurse most specifically expect to note in the infant? 1.Excessive oral secretions 2.Bowel sounds heard over the chest 3.Hiccups and spitting up after a meal 4.Coughing, wheezing, and short periods of apnea

4.Coughing, wheezing, and short periods of apnea

The nurse is reviewing the record of a newborn infant in the nursery and notes that the health care provider has documented the presence of a cephalohematoma. Based on this documentation, what should the nurse expect to note on assessment of the infant? 1.A suture split greater than 1 cm 2.A hard, rigid, immobile suture line 3.Swelling of the soft tissues of the head and scalp 4.Edema resulting from bleeding below the periosteum of the cranium

4.Edema resulting from bleeding below the periosteum of the cranium

An infant is born to a mother with hepatitis B. Which prophylactic measure would be indicated for the infant? 1.Hepatitis B vaccine given within 24 hours after birth 2.Immune globulin (IG) given as soon as possible after delivery 3.Hepatitis B immune globulin (HBIG) given within 14 days after birth 4.Hepatitis B immune globulin (HBIG) and hepatitis B vaccine given within 12 hours after birth

4.Hepatitis B immune globulin (HBIG) and hepatitis B vaccine given within 12 hours after birth

The staff nurse in a neonatal intensive care unit is aware that red electrical outlets denote emergency power and will function in the event of an outage. There are only two red outlets in the room of a 4-day-old male newborn being treated for physiological jaundice and to rule out sepsis from group B streptococcal exposure. Which pieces of equipment requiring power would the nurse select to be plugged into the red outlets in case of a power outage? Select all that apply. 1.Call bell 2.Feeding pump 3.Vital sign machine 4.Phototherapy lights 5.Intravenous (IV) pump

4.Phototherapy lights 5.Intravenous (IV) pump

9. The nurse is caring for a newborn that is being breastfed. Two days following birth, the nurse would expect the stool color to be: a. yellow. b. brown. c. greenish brown. d. black and tarry.

ANS: A The stool of a breastfed infant is bright yellow, soft, and pasty.

13. The mother of a 2-week-old infant tells the nurse that she thinks he is sleeping too much. The most appropriate nursing response to this mother would be: a. "Tell me how many hours per day your baby sleeps." b. "It is normal for newborns to sleep most of the day." c. "Newborns generally sleep 12 to 15 hours per day." d. "You will find as the baby gets older, he sleeps less."

ANS: A While it is true that newborns sleep a great deal of any 24-hour period, the nurse must find out what the mother means by "too much" before giving any information.

26. The nurse is aware that a full-term infant is born with which reflex(es)? Select all that apply. a. Blinking b. Sneezing c. Gagging d. Sucking e. Grasping

ANS: A, B, C, D, E All listed reflexes are present in the full-term newborn.

24. The nurse reminds new parents that newborns must be protected from environments that are too cold or too hot because of which aspect(s) of the newborn's physiology? Select all that apply. a. Very little subcutaneous fat b. Low metabolic rates c. Ineffective sweat glands d. Small fluid reserves e. Low red blood cells counts

ANS: A, C Newborns have very little subcutaneous fat, which offers little insulation against cold. Newborns have ineffective sweat glands and cannot cool themselves through evaporation.

25. Which intervention(s) would be included in the nursing care of the newly circumcised infant? Select all that apply. a. Wash penis with warm water. b. Wipe with alcohol swab. c. Gently remove the yellow crust formation. d. Apply diaper loosely. e. Dress with simple bandage.

ANS: A, D Postcircumcision care includes washing with warm water, avoiding alcohol wipes, leaving the yellow crust in place, and diapering loosely.

11. A full-term newborn weighs 3600 grams at birth. When he is weighed 3 days later, the nurse would expect this newborn to weigh _____ grams. a. 2900 b. 3100 c. 3300 d. 3800

ANS: C In the first 3 to 4 days of life, a newborn generally loses 5% to 10% of his or her birth weight.

18. The nurse is going to use a bulb syringe to clear mucus from a newborn's nose and mouth. The nurse's first action is to: a. place the tip in the nose and squeeze the bulb gently. b. suction secretions from the nose before the mouth. c. depress the bulb before inserting the syringe tip into the mouth. d. insert the tip into the back of the mouth to reach mucus

ANS: C The bulb is depressed, and then the tip is first inserted into the mouth and then the nose. The depression is slowly released, creating the suction.

7. The statement that indicates the parent understands the guidelines for bathing a newborn is: a. "I'll use a mild soap to clean all of the body parts." b. "I am going to add bath oil to the water to keep the baby's skin soft." c. "I should shampoo the head after washing the rest of the body." d. "I'll wash from the feet upward and change the wash cloth for the face."

ANS: C The shampoo is done last because the large surface area of the head predisposes the infant to heat loss

2. The nurse's best response to a mother who is voicing concern about the molding of her 2-day-old infant is: a. "Molding doesn't cause any problems. Don't worry about it." b. "Did you deliver vaginally or by cesarean section?" c. "The baby's head conformed to the shape of the birth canal. It will go away soon." d. "A traumatic delivery can cause molding."

ANS: C The shape of the newborn's head may be out of shape from molding. This refers to the shaping of the fetal head to conform to the size and shape of the birth canal.

17. Parents express concern about the milia on the face and nose of their infant. The nurse's most helpful response would be to instruct the parents to: a. contact a pediatric dermatologist for topical medication. b. squeeze out the white material after cleansing the face. c. wash the infant's face with a mild astringent several times a day. d. leave the milia alone; it will disappear spontaneously. No treatment is needed.

ANS: D Milia require no treatment. This skin manifestation will disappear spontaneously.

19. The mother of a 4-day-old calls the pediatrician's office because she is concerned about her infant's skin. The finding that needs to be reported promptly to the child's pediatrician is: a. the hands and feet feel cooler than the rest of the body. b. skin is peeling on several parts of the infant's body. c. there is a small pink patch on the left eyelid and one on the neck. d. today, the infant's skin has a yellowish tinge.

ANS: D Physiological jaundice becomes evident between the second and third days of life and lasts for about 1 week. Evidence of jaundice is reported and the newborn is evaluated.

10. The mother of a 2-week-old infant tells the nurse, "I think the baby is constipated. I've noticed she strains when she has a bowel movement." The nurse's most helpful response would be: a. "Give the baby one serving of fruit per day." b. "Increase the amount and frequency of her feedings." c. "It sounds like the baby is uncomfortable because she is constipated." d. "Newborns might strain with bowel movements because their muscles aren't fully developed."

ANS: D Straining in the newborn period is normal. It results from underdeveloped abdominal musculature. No treatment is required.

A client in labor for the past 10 hours shows no change in cervical dilation and has stayed at 5 to 6 cm for the past 2 hours. Her contractions remain regular at 2-minute intervals, lasting 40 to 45 seconds. Which of the following would be the nurse's initial action? 1. Assess for presence of a full bladder. 2. Suggest the placement of an internal uterine pressure catheter to determine adequacy of contractions. 3. Encourage the mother to relax by assisting her with appropriate breathing techniques. 4. Suggest to the physician that oxytocin augmentation be started to stimulate labor.

Answer: 1 RATIONALES: A full bladder will slow or stop cervical dilation and produce symptoms that could be misdiagnosed as arrest in labor. Other strategies, such as internal uterine monitoring, relaxation, and oxytocin augmentation, would be appropriate later, but assessing the bladder first is key.

The nurse observes a late deceleration. It's characterized by and indicates which of the following? 1. U-shaped deceleration occurring after the first half of the contraction, indicating uteroplacental insufficiency 2. U-shaped deceleration occurring with the contraction, indicating cord compression 3. V-shaped deceleration occurring after the contraction, indicating uteroplacental insufficiency 4. Deep U-shaped deceleration occurring before the contraction, indicating head compression

Answer: 1 RATIONALES: A late deceleration is U-shaped and occurs after the first half of the contraction, indicating uteroplacental insufficiency. It's an ominous pattern and requires immediate action — such as administering oxygen, repositioning the mother, and increasing the I.V. infusion rate — to correct the problem. U- and V-shaped decelerations are variable decelerations occurring at unpredictable times during contractions and are related to umbilical cord compression. Deep U-shaped deceleration occurring before the contraction is early deceleration.

For a client who's moving into the active phase of labor, the nurse should include which of the following as the priority of care? 1. Offer support by reviewing the short-pant form of breathing. 2. Administer narcotic analgesia. 3. Allow the mother to walk around the unit. 4. Watch for rupture of the membranes.

Answer: 1 RATIONALES: By helping the client use the pant form of breathing, the nurse can help the client manage her contractions and reduce the need for opioids and other forms of pain relief, which can have an effect on fetal outcome. In the active phase, the mother most likely is too uncomfortable to walk around the unit. The nurse will observe for rupture of membranes and may administer opioid analgesia but these don't take priority.

A primigravid client is admitted to the labor and delivery area. Assessment reveals that she's in the early part of the first stage of labor. Her pain is likely to be most intense: 1. around the pelvic girdle. 2. around the pelvic girdle and in the upper legs. 3. around the pelvic girdle and at the perineum. 4. at the perineum.

Answer: 1 RATIONALES: During most of the first stage of labor, pain centers around the pelvic girdle. During the late part of this stage and the early part of the second stage, pain spreads to the upper legs and perineum. During the late part of the second stage and during childbirth, intense pain occurs at the perineum.

Assessment of a client in active labor reveals meconium-stained amniotic fluid and fetal heart sounds in the upper right quadrant. Which of the following is the most likely cause of this situation? 1. Breech position 2. Late decelerations 3. Entrance into the second stage of labor 4. Multiple gestation

Answer: 1 RATIONALES: Fetal heart sounds in the upper right quadrant and meconium-stained amniotic fluid indicate a breech presentation. The staining is usually caused by the squeezing actions of the uterus on a fetus in the breech position, although late decelerations, entrance into the second stage of labor, and multiple gestation may contribute to meconium-stained amniotic fluid.

Which of the following describes the term fetal position? 1. Relationship of the fetus's presenting part to the mother's pelvis 2. Fetal posture 3. Fetal head or breech at cervical os 4. Relationship of the fetal long axis to the mother's long axis

Answer: 1 RATIONALES: Fetal position refers to the relationship of the fetus's presenting part to the mother's pelvis. Fetal posture refers to "attitude." Presentation refers to the part of the fetus at the cervical os. Lie refers to the relationship of the fetal long axis to that of the mother's long axis.

During labor, a client asks the nurse why her blood pressure must be measured so often. Which explanation should the nurse provide? 1. Blood pressure reflects changes in cardiovascular function, which may affect the fetus. 2. Increased blood pressure indicates that the client is experiencing pain. 3. Increased blood pressure signals the peak of the contraction. 4. Medications given during labor affect blood pressure.

Answer: 1 RATIONALES: Frequent blood pressure measurement helps determine whether maternal cardiovascular function is adequate. During contractions, blood flow to the intervillous spaces changes, compromising fetal blood supply. Increased blood pressure is expected during pain and contractions. Measuring blood pressure frequently helps determine whether blood pressure has returned to precontraction levels, ensuring adequate blood flow to the fetus. Although medications given during labor can affect blood pressure, the main purpose of measuring blood pressure is to verify adequate fetal status.

When assessing a client who has just delivered a neonate, the nurse finds that the fundus is boggy and deviated to the right. What should the nurse do? 1. Have the client void. 2. Assess the client's vital signs. 3. Evaluate lochia characteristics. 4. Massage the fundus.

Answer: 1 RATIONALES: Having the client void can determine whether the boggy, deviated fundus results from a full bladder — the most common cause of these fundal findings. Vital sign assessment is unnecessary unless the nurse suspects hemorrhage from delayed involution. In a client who doesn't have a full bladder, the nurse should evaluate lochia characteristics to detect possible hemorrhage. If the client has a full bladder, massaging the fundus won't stimulate uterine contractions (which aid uterine involution) or prevent uterine atony — a possible cause of hemorrhage.

The nurse applies an external electronic fetal monitor (EFM) to assess a client's uterine contractions and evaluate the fetal heart rate (FHR). However, the client is uncomfortable and changes positions frequently, making FHR hard to assess. Consequently, the physician decides to switch to an internal EFM. Before internal monitoring can begin, which of the following must occur? 1. The membranes must rupture. 2. The client must receive anesthesia. 3. The cervix must be fully dilated. 4. The fetus must be at 0 station.

Answer: 1 RATIONALES: Internal EFM can be used only after the client's membranes rupture, when the cervix is dilated at least 2 cm and when the presenting part is at least at -1 station. Anesthesia isn't required for internal EFM.

A client, age 19, goes into labor at 40 weeks' gestation. When assessing the fetal monitor strip, the nurse sees that the fetal heart rate (FHR) has decreased to 60 beats/minute and that the waveforms sometimes resemble a V and begin and end abruptly. The nurse should interpret this pattern as: 1. variable decelerations. 2. decreased short-term variability. 3. increased long-term variability. 4. early decelerations.

Answer: 1 RATIONALES: On a fetal monitor strip, variable decelerations are characterized by an FHR that commonly decreases to 60 beats/minute; waveform shapes that vary and may resemble the letter U, V, or W; and deceleration waveforms with an abrupt onset and recovery. Decreased short-term variability manifests as fewer than 2 to 3 beats/amplitude of the baseline FHR. Increased long-term variability manifests as more than 5 to 20 beats/minute of the baseline FHR in rhythmic fluctuation. Early decelerations are seen as the descent, peak, and recovery of the deceleration waveform that mirrors the contraction waveform.

During labor, a client's cervix fails to dilate progressively, despite her uncomfortable uterine contractions. To augment labor, the physician orders oxytocin (Pitocin). When preparing the client for oxytocin administration, the nurse describes the contractions the client is likely to feel when she starts to receive the drug. Which description is accurate? 1. Contractions will be stronger and more uncomfortable and will peak more abruptly. 2. Contractions will be weaker, longer, and more effective. 3. Contractions will be stronger, shorter, and less uncomfortable. 4. Contractions will be stronger and shorter and will peak more slowly.

Answer: 1 RATIONALES: Oxytocin administration causes stronger, more uncomfortable contractions, which peak more abruptly than spontaneous contractions. Oxytocin doesn't affect the duration of contractions.

The nurse is administering oxytocin (Pitocin) to a client in labor. During oxytocin therapy, why must the nurse monitor the client's fluid intake and output closely? 1. Because oxytocin causes fluid retention 2. Because oxytocin causes excessive thirst 3. Because oxytocin has a diuretic effect

Answer: 1 RATIONALES: Oxytocin has an antidiuretic effect; prolonged I.V. infusion may lead to severe fluid retention, resulting in seizures, coma, and even death. Excessive thirst results from the work of labor and lack of oral fluids, not oxytocin administration. Oxytocin isn't toxic to the kidney.

At 28 weeks' gestation, a client is admitted to the labor and delivery area in preterm labor. An I.V. infusion of ritodrine (Yutopar) is started. Which client outcome reflects the nurse's awareness of an adverse effect of ritodrine? 1. "The client remains free from tachycardia." 2. "The client remains free from polyuria." 3. "The client remains free from hypertension." 4. "The client remains free from hyporeflexia."

Answer: 1 RATIONALES: Ritodrine and other beta-adrenergic agonists may cause tachycardia, hypotension, bronchial dilation, increased plasma volume, increased cardiac output, arrhythmias, myocardial ischemia, reduced urine output, restlessness, headache, nausea, and vomiting. These drugs aren't associated with polyuria, hypertension, or hyporeflexia.

A client is admitted to the labor and delivery department in preterm labor. To help manage preterm labor the nurse would expect to administer: 1. ritodrine (Yutopar). 2. bromocriptine (Parlodel). 3. magnesium sulfate. 4. betamethasone (Celestone).

Answer: 1 RATIONALES: Ritodrine reduces frequency and intensity of uterine contractions by stimulating B2 receptors in the uterine smooth muscle. It's the drug of choice when trying to inhibit labor. Bromocriptine, a dopamine receptor agonist and an ovulation stimulant, is used to inhibit lactation in the postpartum period. Magnesium sulfate, an anticonvulsant, is used to treat preeclampsia and eclampsia — a life-threatening form of pregnancy-induced hypertension. Betamethasone, a synthetic corticosteroid, is used to stimulate fetal pulmonary surfactant (administered to the mother).

A diabetic client in labor tells the nurse she has had trouble controlling her blood glucose level recently. She says she didn't take her insulin when the contractions began because she felt nauseated; about an hour later, when she felt better, she ate some soup and crackers but didn't take insulin. Now, she reports increased nausea and a flushed feeling. The nurse notes a fruity odor to her breath. What do these findings suggest? 1. Diabetic ketoacidosis 2. Hypoglycemia 3. Infection 4. Transition to the active phase of labor

Answer: 1 RATIONALES: Signs and symptoms of diabetic ketoacidosis include nausea and vomiting, a fruity or acetone breath odor, signs of dehydration (such as flushed, dry skin), hyperglycemia, ketonuria, hypotension, deep and rapid respirations, and a decreased level of consciousness. In contrast, hypoglycemia causes sweating, tremors, palpitations, and behavioral changes. Infection causes a fever. Transition to the active phase of labor is signaled by cervical dilation of up to 7 cm and contractions every 2 to 5 minutes.

Because cervical effacement and dilation aren't progressing in a client in labor, the physician orders I.V. administration of oxytocin (Pitocin). Why must the nurse monitor the client's fluid intake and output closely during oxytocin administration? 1. Oxytocin causes water intoxication. 2. Oxytocin causes excessive thirst. 3. Oxytocin is toxic to the kidneys. 4. Oxytocin has a diuretic effect

Answer: 1 RATIONALES: The nurse should monitor fluid intake and output because prolonged oxytocin infusion may cause severe water intoxication, leading to seizures, coma, and death. Excessive thirst results from the work of labor and limited oral fluid intake — not oxytocin. Oxytocin has no nephrotoxic or diuretic effects. In fact, it produces an antidiuretic effect.

When assessing the fetal heart rate tracing, the nurse becomes concerned about the fetal heart rate pattern. In response to the loss of variability, the nurse repositions the client to her left side and administers oxygen. These actions are likely to improve which of the following? 1. Fetal hypoxia 2. The contraction pattern 3. The status of a trapped cord 4. Maternal comfort

Answer: 1 RATIONALES: These actions, which will improve fetal hypoxia, increase the amount of maternal circulating oxygen by taking pressure created by the uterus off the aorta and improving blood flow. These actions won't improve the contraction pattern, free a trapped cord, or improve maternal comfort.

The nurse is evaluating a client who is 34 weeks pregnant for premature rupture of the membranes (PROM). Which findings indicate that PROM has occurred? 1. Fernlike pattern when vaginal fluid is placed on a glass slide and allowed to dry 2. Acidic pH of fluid when tested with nitrazine paper 3. Presence of amniotic fluid in the vagina 4. Cervical dilation of 6 cm 5. Alkaline pH of fluid when tested with nitrazine paper

Answer: 1,3,5 RATIONALES: The fernlike pattern that occurs when vaginal fluid is placed on a glass slide and allowed to dry, presence of amniotic fluid in the vagina, and alkaline pH of fluid are all signs of ruptured membranes. The fernlike pattern seen when the fluid is allowed to dry on a slide is a result of the high sodium and protein content of the amniotic fluid. The presence of amniotic fluid in the vagina results from the expulsion of the fluid from the amniotic sac. Cervical dilation and regular contractions are signs of progressing labor but don't indicate PROM.

The nurse is monitoring a client who is receiving oxytocin (Pitocin) to induce labor. The nurse should be prepared for which maternal adverse reactions? 1. Hypertension 2. Jaundice 3. Dehydration 4. Fluid overload 5. Uterine tetany 6. Bradycardia

Answer: 1,4,5 RATIONALES: Adverse reactions to oxytocin in the mother include hypertension, fluid overload, and uterine tetany. The antidiuretic effect of oxytocin increases renal reabsorption of water, leading to fluid overload — not dehydration. Jaundice and bradycardia are adverse reactions that may occur in the neonate. Tachycardia, not bradycardia, is reported as a maternal adverse reaction.

The nurse notices that a client in the first stage of labor seems agitated. When the nurse asks why she's upset, she begins to cry and says, "I guess I'm a little worried. The last time I gave birth, I was in labor for 32 hours." Based on this information, the nurse should include which nursing diagnosis in the client's care plan? 1. Anxiety related to the facility environment 2. Fear related to a potentially difficult childbirth 3. Compromised family coping related to hospitalization 4. Acute pain related to labor contractions

Answer: 2 RATIONALES: A client's ability to cope during labor and delivery may be hampered by fear of a painful or difficult childbirth, fear of loss of control or self-esteem during childbirth, or fear of fetal death. A previous negative experience may increase these fears. Therefore, Fear related to a potentially difficult childbirth is the most appropriate nursing diagnosis. The client's anxiety stems from her past history of a long labor, not from being in the facility; therefore a diagnosis of Anxiety related to the facility environment isn't warranted. There is no evidence of compromised family coping related to hospitalization. Although acute pain related to labor contractions may be a problem, this isn't mentioned in the question.

The physician decides to artificially rupture the membranes. Following this procedure, the nurse checks the fetal heart tones for which reason? 1. To determine fetal well-being 2. To assess for fetal bradycardia 3. To assess fetal position 4. To prepare for an imminent delivery

Answer: 2 RATIONALES: After a client has an amniotomy, the nurse should assure that the cord isn't prolapsed and that the baby tolerated the procedure well. The most effective way to do this is to check the fetal heart rate. Fetal well-being is assessed via a nonstress test. Fetal position is determined by vaginal examination. Artificial rupture of membranes doesn't indicate an imminent delivery.

During the active phase of the first stage of labor, a client undergoes an amniotomy. After this procedure, which nursing diagnosis takes the highest priority? 1. Deficient knowledge (testing procedure) related to amniotomy 2. Ineffective fetal cerebral tissue perfusion related to cord compression 3. Acute pain related to increasing strength of contractions 4. Risk for infection related to rupture of membranes

Answer: 2 RATIONALES: Amniotomy increases the risk of cord prolapse. If the prolapsed cord is compressed by the presenting fetal part, the fetal blood supply may be impaired, jeopardizing the fetal oxygen supply. Because lack of oxygen to the fetus may cause fetal death, the nursing diagnosis of Ineffective fetal cerebral tissue perfusion takes priority over diagnoses of Deficient knowledge, Acute pain, and Risk for infection.

A client with Rh isoimmunization delivers a neonate with an enlarged heart and severe, generalized edema. Which nursing diagnosis is most appropriate for this client? 1. Ineffective denial related to a socially unacceptable infection 2. Impaired parenting related to the neonate's transfer to the intensive care unit 3. Deficient fluid volume related to severe edema 4. Fear related to removal and loss of the neonate by statute

Answer: 2 RATIONALES: Because the neonate is severely ill and needs to be placed in the neonatal intensive care unit, the client may have a nursing diagnosis of Impaired parenting related to the neonate's transfer to the neonatal intensive care unit. (Another pertinent nursing diagnosis may be Compromised family coping related to lack of opportunity for bonding.) Rh isoimmunization isn't a socially unacceptable infection. This condition causes an excess fluid volume (not deficient) related to cardiac problems. Rh isoimmunization doesn't lead to loss of the neonate by statute.

After admission to the labor and delivery area, a client undergoes routine tests, including a complete blood count, urinalysis, Venereal Disease Research Laboratory test, and gonorrhea culture. The gonorrhea culture is positive, although the client lacks signs and symptoms of this disease. What is the significance of this finding? 1. Maternal gonorrhea may cause a neural tube defect in the fetus. 2. Maternal gonorrhea may cause an eye infection in the neonate. 3. Maternal gonorrhea may cause acute liver changes in the fetus. 4. Maternal gonorrhea may cause anemia in the neonate.

Answer: 2 RATIONALES: Gonorrhea in the cervix may cause neonatal eye infection during delivery as well as a serious puerperal infection in the client. Maternal gonorrhea isn't associated with neural tube defects, acute fetal liver changes, or neonatal anemia.

A primigravid client, age 20, has just completed a difficult, forceps-assisted delivery of twins. Her labor was unusually long and required oxytocin (Pitocin) augmentation. The nurse who's caring for her should stay alert for: 1. uterine inversion. 2. uterine atony. 3. uterine involution. 4. uterine discomfort.

Answer: 2 RATIONALES: Multiple fetuses, extended labor stimulation with oxytocin, and traumatic delivery commonly are associated with uterine atony, which may lead to postpartum hemorrhage. Uterine inversion may precede or follow delivery and commonly results from apparent excessive traction on the umbilical cord and attempts to deliver the placenta manually. Uterine involution and some uterine discomfort are normal after delivery.

At 40 weeks' gestation, a client is admitted to the labor and delivery area. She and her husband are worried about the fetus's health because she had problems during her previous childbirth. The nurse reassures them that the fetus will be monitored closely with an electronic fetal monitor (EFM). On the fetal monitor strip, what is the single most reliable indicator of fetal well-being? 1. Normal long-term variability 2. Normal short-term variability 3. Normal baseline fetal heart rate (FHR) 4. Normal contraction sequence

Answer: 2 RATIONALES: Normal short-term variability — 2 to 3 beats per amplitude — is the single most reliable indicator of fetal well-being on an EFM strip. It represents actual beat-to-beat fluctuations in the FHR. Normal long-term variability, although a helpful indicator, takes into account larger periodic and rhythmic deviations above and below the baseline FHR. Baseline FHR serves only as a reference for all subsequent FHR readings taken during labor. Contraction sequence provides no information about fetal well-being, although it does give some indication of maternal well-being and progress.

The nurse notices that a large number of clients who receive oxytocin (Pitocin) to induce labor, vomit as the infusion is started. The nurse assesses the situation further and discovers that these clients received no instruction before arriving on the unit and haven't fasted for 8 hours before induction. How should the nurse intervene? 1. Notify the physicians and explain that they need to teach their clients before inducing labor. 2. Initiate a unit policy involving staff nurses, certified nurse midwives, and physicians in teaching clients before labor induction. 3. Report the physicians for providing inferior care. 4. Initiate a protocol order that allows the nurse to administer promethazine (Phenergan) before administering oxytocin.

Answer: 2 RATIONALES: The best intervention by the nurse is to initiate a unit policy that involves the multidisciplinary team. This approach creates an atmosphere of collegiality and professionalism with the goal of providing the best care for clients in labor. Option 1 blames the physician and doesn't promote multidisciplinary teamwork. Reporting the physicians is unnecessary because nothing indicates that the physicians provided inferior care. The nurse can approach the medical staff about initiating a protocol order that allows the nursing staff to administer promethazine; however, this option doesn't address the problem — the lack of client education.

A pregnant client arrives at the health care facility, stating that her bed linens were wet when she woke up this morning. She says no fluid is leaking but complains of mild abdominal cramps and lower back discomfort. Vaginal examination reveals cervical dilation of 3 cm, 100% effacement, and positive ferning. Based on these findings, the nurse concludes that the client is in which phase of the first stage of labor? 1. Active phase 2. Latent phase 3. Expulsive phase 4. Transitional phase

Answer: 2 RATIONALES: The latent phase of the first stage of labor is associated with irregular, short, mild contractions; cervical dilation of 3 to 4 cm; and abdominal cramps or lower back discomfort. During the active phase, the cervix dilates to 7 cm and moderately intense contractions of 40 to 50 seconds' duration occur every 2 to 5 minutes. Fetal descent continues throughout the active phase and into the transitional phase, when the cervix dilates from 8 to 10 cm and intense contractions of 45 to 60 seconds' duration occur every 1½ to 2 minutes. The first stage of labor doesn't include an expulsive phase.

A client is admitted to the labor and delivery area. How can the nurse most effectively determine the duration of the client's contractions? 1. By timing the period between one contraction and the beginning of the next contraction 2. By timing the period from the onset of uterine tightening to uterine relaxation 3. By timing the period from the increment (building-up) phase to the acme (peak) phase 4. By timing the period from the acme (peak) phase to the decrement (letting-down) phase

Answer: 2 RATIONALES: To determine the duration of contractions, the nurse should time the period from the onset of uterine tightening to uterine relaxation. Timing the period between one contraction and the beginning of the next contraction helps determine the frequency of contractions. Timing the period from the increment to the acme or from the acme to the decrement supplies only partial information about contractions.

An adolescent in the early stages of labor is admitted to the labor and delivery unit. The nurse notes lymphadenopathy and a macular rash on the palmar surfaces of the hands and plantar surfaces of the feet. Admission laboratory testing reveals trace ketones in the urine, white blood cell count 10,000/μl, hemoglobin 14.5 g/dl, hematocrit 40%, and the nontreponemal antibody test is positive. The nurse notifies the physician of the laboratory results. Which action by the nurse takes priority? 1. Notifying the laboratory that a repeat hemoglobin and hematocrit have been ordered. 2. Recommending that the client drink plenty of fluids. 3. Consulting with the infection control nurse. 4. Asking the client if she has been exposed to varicella in the past 3 weeks.

Answer: 3 RATIONALES: A nontreponemal test screens the client for syphilis. The positive test result, along with the lymphadenopathy and rash, indicate that the client has secondary syphilis. Based on these findings, the neonate will most likely have signs and symptoms of congenital syphilis. The hemoglobin and hematocrit results are normal for a pregnant client. The laboratory results don't show signs of dehydration, so having the client drink plenty of fluids isn't necessary. The lesions associated with varicella are vesicular, and don't resemble the rash associated with syphilis.

A client with intrauterine growth retardation is admitted to the labor and delivery unit and started on an I.V. infusion of oxytocin (Pitocin). Which of the following is least likely to be included in her care plan? 1. Carefully titrating the oxytocin based on her pattern of labor 2. Monitoring vital signs, including assessment of fetal well-being, every 15 to 30 minutes 3. Allowing the client to ambulate as tolerated 4. Helping the client use breathing exercises to manage her contractions

Answer: 3 RATIONALES: Because the fetus is at risk for complications, frequent and close monitoring is necessary. Therefore, the client shouldn't be allowed to ambulate. Carefully titrating the oxytocin, monitoring vital signs, including fetal well-being, and assisting with breathing exercises are appropriate actions to include.

The nurse is assessing the fetal heart rate (FHR) of a client, who is at term, admitted to the labor and delivery area. Which of the following should the nurse identify as the normal range of the baseline FHR? 1. 60 to 80 beats/minute 2. 80 to 120 beats/minute 3. 120 to 160 beats/minute 4. 160 to 200 beats/minute

Answer: 3 RATIONALES: In a full-term fetus, the baseline FHR normally ranges from 120 to 160 beats/minute. FHR below 120 beats/minute reflects bradycardia; above 160 beats/minute, tachycardia.

Which of the following would be an inappropriate indication of placental detachment? 1. An abrupt lengthening of the cord 2. An increase in the number of contractions 3. Relaxation of the uterus 4. Increased vaginal bleeding

Answer: 3 RATIONALES: Relaxation isn't an indication for detachment of the placenta. An abrupt lengthening of the cord, an increase in the number of contractions, and an increase in vaginal bleeding are all indications that the placenta has detached from the wall of the uterus.

The third stage of labor ends with which of the following? 1. The birth of the baby 2. When the client is fully dilated 3. After the delivery of the placenta 4. When the client is transferred to her postpartum bed

Answer: 3 RATIONALES: The definition of the third stage of labor is the delivery of the placenta. The first stage of labor ends with complete cervical dilation and effacement. The second stage of labor ends with the birth of the baby. The fourth stage of labor includes the first 4 hours after birth.

During labor, a client tells the nurse that her last baby "came out really fast." The nurse can help control a precipitous delivery by: 1. applying counterpressure to the fetus's head. 2. encouraging the client to push. 3. massaging and supporting the perineum. 4. instructing the client to contract the perineal muscles.

Answer: 3 RATIONALES: The nurse can help control a precipitous delivery by stretching the labia, such as by massaging and bracing the perineum with gentle back pressure. This helps prevent perineal lacerations — the primary maternal complication of precipitous delivery. Applying counterpressure to the fetus's head reduces perineal stress temporarily; however, delivery proceeds when the client pushes with uterine contractions. Pushing puts further stress on the perineum, promoting delivery. When the fetus's head exerts pressure on the perineum, contracting the perineal muscles is virtually impossible.

A nursing assistant escorts a client in the early stages of labor to the bathroom. When the nurse enters the client's room, she detects a strange odor coming from the bathroom and suspects the client has been smoking marijuana. What should the nurse do next? 1. Tell the client that smoking is prohibited in the facility, and that if she smokes again, she'll be discharged. 2. Explain to the client that smoking poses a danger of explosion because oxygen tanks are stored close by. 3. Notify the physician and security immediately. 4. Ask the nursing assistant to dispose of the marijuana that the client can't smoke anymore.

Answer: 3 RATIONALES: The nurse should immediately notify the physician and security. The physician must be informed because illegal drugs can interfere with the labor process and affect the neonate after delivery. Moreover, the client might have consumed other illegal drugs. The nurse should also inform security who are specially trained to handle such situations. Most hospitals prohibit smoking. The nurse needs to alert others about the client's illegal drug use, not simply explain to the client that smoking is prohibited. Smoking is dangerous around oxygen and it's fine for the nurse to explain the hazard to the client; however, the nurse must first notify the physician and security. The nursing assistant shouldn't be asked to dispose of the marijuana.

A client in the fourth stage of labor asks to use the bathroom for the first time since delivery. The client has oxytocin (Pitocin) infusing. Which response by the nurse is best? 1. "You'll have to wait until the vaginal bleeding stops." 2. "You'll have to wait until the oxytocin is infused." 3. "You may use the bathroom with my assistance." 4. "You may get up to the bathroom whenever you need to."

Answer: 3 RATIONALES: The nurse should tell the client that she may use the bathroom with the nurse's assistance. The nurse should assist the client for the client's first trip to the bathroom after delivery. It isn't uncommon for a client to faint after delivery. Telling the client she must wait until her vaginal bleeding stops is inappropriate; vaginal bleeding continues for about 6 weeks after delivery. The nurse shouldn't tell the client she can get up whenever she needs to use the bathroom; doing so places the client at risk for injury.

When caring for a client with preeclampsia, which action is a priority? 1. Monitoring the client's labor carefully and preparing for a fast delivery 2. Continually assessing the fetal tracing for signs of fetal distress 3. Checking vital signs every 15 minutes to watch for increasing blood pressure 4. Reducing visual and auditory stimulation

Answer: 4 RATIONALES: A client with preeclampsia is at risk for seizure activity because her neurologic system is overstimulated. Therefore, in addition to administering pharmacologic interventions to reduce the possibility of seizures, the nurse should lessen auditory and visual stimulation. Although the other actions are important, they're of a lesser priority.

A nulliparous client has been in the latent phase of the first stage of labor for several hours. Despite continued uterine contractions, her cervix hasn't dilated further since the initial examination. Her latent phase may be considered prolonged after: 1. 6 hours. 2. 10 hours. 3. 14 hours. 4. 20 hours.

Answer: 4 RATIONALES: Based on research, the latent phase may be considered prolonged if it exceeds 20 hours in a nulliparous client or 14 hours in a multiparous client.

A client with hemolysis, elevated liver enzymes, and low platelet count (HELLP) syndrome is admitted to the labor and delivery unit. The client's condition rapidly deteriorates and despite efforts by the staff, the client dies. After the client's death, the nursing staff displays many emotions. Who should the nurse manager consult to help the staff cope with this unexpected death? 1. The human resource director, so she can arrange vacation time for the staff 2. The physician, so he can provide education about HELLP syndrome 3. The social worker, so she can contact the family about funeral arrangements and pass along the information to the nursing staff 4. The chaplain, because his educational background includes strategies for handling grief

Answer: 4 RATIONALES: The chaplain should be consulted because his educational background provides strategies for helping others handle grief. Providing the staff with vacation isn't feasible from a staffing standpoint and doesn't help staff cope with their grief. The staff needs grief counseling, not education about HELLP syndrome. Asking the social worker to contact the family about the funeral arrangements isn't appropriate.

The nurse is caring for a client who's in the first stage of labor. What is the shortest but most difficult part of this stage? 1. Active phase 2. Complete phase 3. Latent phase 4. Transitional phase

Answer: 4 RATIONALES: The transitional phase, which lasts 1 to 3 hours, is the shortest but most difficult part of the first stage of labor. This phase is characterized by intense uterine contractions that occur every 1½ to 2 minutes and last 45 to 90 seconds. The active phase lasts 4½ to 6 hours; it's characterized by contractions that start out moderately intense, grow stronger, and last about 60 seconds. The complete phase occurs during the second, not first, stage of labor. The latent phase lasts 5 to 8 hours and is marked by mild, short, irregular contractions.

A PP client is being treated for DVT. The nurse understands that the client's response to treatment will be evaluated by regularly assessing the client for: 1. Dysuria, ecchymosis, and vertigo 2. Epistaxis, hematuria, and dysuria 3. Hematuria, ecchymosis, and epistaxis 4. Hematuria, ecchymosis, and vertigo

3. The treatment for DVT is anticoagulant therapy. The nurse assesses for bleeding, which is an adverse effect of anticoagulants. This includes hematuria, ecchymosis, and epistaxis. Dysuria and vertigo are not associated specifically with bleeding.

A nurse is assessing a client in the 4th stage if labor and notes that the fundus is firm but that bleeding is excessive. The initial nursing action would be which of the following? 1. Massage the fundus 2. Place the mother in the Trendelenburg's position 3. Notify the physician 4. Record the findings

3. If the bleeding is excessive, the cause may be laceration of the cervix or birth canal. Massaging the fundus if it is firm will not assist in controlling the bleeding. Trendelenburg's position is to be avoided because it may interfere with cardiac function.

After expulsion of the placenta in a client who has six living children, an infusion of lactated ringer's solution with 10 units of pitocin is ordered. The nurse understands that this is indicated for this client because: 1. She had a precipitate birth 2. This was an extramural birth 3. Retained placental fragments must be expelled 4. Multigravida's are at increased risk for uterine atony.

4. Multiple full-term pregnancies and deliveries result in overstretched uterine muscles that do not contract efficiently and bleeding may ensue.

The nurse is admitting a newborn infant to the nursery and notes that the health care provider has documented that the newborn has an omphalocele. While performing an assessment, where should the nurse document the location of the viscera in this condition? 1.Inside the abdominal cavity and under the skin 2.Inside the abdominal cavity and under the dermis 3.Outside the abdominal cavity and not covered with a sac 4. Outside the abdominal cavity but inside a translucent sac covered with peritoneum and amniotic membrane

4. Outside the abdominal cavity but inside a translucent sac covered with peritoneum and amniotic membrane

23. What noninvasive form(s) of pain relief might a nurse implement with a newborn? Select all that apply. a. Swaddling b. Rocking c. Offering a pacifier d. Distraction e. Cuddling

ANS: A, B, C, E Swaddling, rocking, nonnutritive sucking, quiet environment, and cuddling are all effective, noninvasive pain remedies. Distraction is not a dependable method of pain reduction with infants.

27. The nurse takes into consideration that newborns are especially prone to dehydration because of which aspect(s) of their physiology? Select all that apply. a. Small glomeruli b. Minimal renal blood flow c. Inactive gastrointestinal (GI) tract d. Excessive fluid loss from the sweat glands e. Immature renal tubules that do not concentrate urine

ANS: A, B, E The newborn's glomeruli are small and have only one third of the blood circulation of an adult and they are unable to effectively concentrate urine. The GI tract is active. The infant's sweat glands do not work effectively and allow very little fluid loss through sweat.

15. On what knowledge would the nurse base a response to a mother who questions, "Do you think my baby recognizes my voice?" a. Voice recognition is delayed because the ears are not well developed at birth. b. Infants respond to voice by increasing movements and sucking. c. Infants initially respond to low-pitched voices. d. Neonates can distinguish a mother's voice from other sounds in the first days of life.

ANS: D The ability to discriminate between a mother's voice and other voices may occur as early as in the first 3 days of life.

6. While assessing the head of a healthy, full-term newborn, the nurse anticipates that the anterior fontanelle is: a. depressed and sunken. b. triangular shaped. c. smaller than the posterior fontanelle. d. open and diamond shaped.

ANS: D The anterior fontanelle is diamond shaped and located at the junction of the two parietal and two frontal bones. It should not be raised or sunken, and it closes between 12 and 18 months of age.

Select all of the physiological maternal changes that occur during the PP period. 1.Cervical involution ceases immediately 2.Vaginal distention decreases slowly 3.Fundus begins to descend into the pelvis after 24 hours 4.Cardiac output decreases with resultant tachycardia in the first 24 hours 5.Digestive processes slow immediately.

1 and 3. In the PP period, cervical healing occurs rapidly and cervical involution occurs. After 1 week the muscle begins to regenerate and the cervix feels firm and the external os is the width of a pencil. Although the vaginal mucosa heals and vaginal distention decreases, it takes the entire PP period for complete involution to occur and muscle tone is never restored to the pregravid state. The fundus begins to descent into the pelvic cavity after 24 hours, a process known as involution. Despite blood loss that occurs during delivery of the baby, a transient increase in cardiac output occurs. The increase in cardiac output, which persists about 48 hours after childbirth, is probably caused by an increase in stroke volume because Bradycardia is often noted during the PP period. Soon after childbirth, digestion begins to begin to be active and the new mother is usually hungry because of the energy expended during labor.

The nurse in the newborn nursery is performing admission vital signs on a newborn infant. The nurse notes that the respiratory rate of the newborn is 50 breaths per minute. Which action should the nurse take? 1. Document the findings. 2. Contact the health care provider. 3. Apply an oxygen mask to the newborn infant. 4. Cover the newborn infant with blankets and reassess the respiratory rate in 15 minutes.

1. Document the findings. The normal respiratory rate for a normal newborn is 30 to 60 breaths per minute.

A nurse is preparing to assess the uterine fundus of a client in the immediate postpartum period. When the nurse locates the fundus, she notes that the uterus feels soft and boggy. Which of the following nursing interventions would be most appropriate initially? 1. Massage the fundus until it is firm 2. Elevate the mothers legs 3. Push on the uterus to assist in expressing clots 4. Encourage the mother to void

1. If the uterus is not contracted firmly, the first intervention is to massage the fundus until it is firm and to express clots that may have accumulated in the uterus. Pushing on an uncontracted uterus can invert the uterus and cause massive hemorrhage. Elevating the client's legs and encouraging the client to void will not assist in managing uterine atony. If the uterus does not remain contracted as a result of the uterine massage, the problem may be distended bladder and the nurse should assist the mother to urinate, but this would not be the initial action.

Which would be considered abnormal findings in a newborn less than 12 hours old? Select all that apply. 1.Grunting respirations 2. Presence of vernix caseosa 3. Heart rate of 190 beats/minute 4. Anterior fontanelle measuring 5.0 cm 5. Bluish discoloration of hands and feet 6. A yellow discoloration of the sclera and body

1.Grunting respirations 3. Heart rate of 190 beats/minute 6. A yellow discoloration of the sclera and body

Which are modes of heat loss in the newborn? Select all that apply. 1.Radiation 2.Urination 3.Convection 4.Conduction 5.Evaporation

1.Radiation 3.Convection 4.Conduction 5.Evaporation

The nurse weighing a term newborn during the initial newborn assessment determines the infant's weight to be 4325 g. The nurse determines that this infant may be at risk for which complications? Select all that apply. 1.Retinopathy 2.Hypoglycemia 3.Fractured clavicle 4.Hyperbilirubinemia 5.Congenital heart defect 6.Necrotizing enterocolitis

2.Hypoglycemia 3.Fractured clavicle 5.Congenital heart defect Any newborn weighing more than 4000 g at birth is defined as being large for gestational age (LGA). Because of their size, LGA infants are also at risk for hypoglycemia. LGA infants also have a higher incidence of birth injuries (fractured clavicle), asphyxia, and congenital anomalies (heart defect).

On which of the postpartum days can the client expect lochia serosa? 1. Days 3 and 4 PP 2. Days 3 to 10 PP 3. Days 10-14 PP 4. Days 14 to 42 PP

2. On the third and fourth PP days, the lochia becomes a pale pink or brown and contains old blood, serum, leukocytes, and tissue debris. This type of lochia usually lasts until PP day 10. Lochia rubra usually last for the first 3 to 4 days PP. Lochia alba, which contain leukocytes, decidua, epithelial cells, mucus, and bacteria, may continue for 2 to 6 weeks PP.

The nurse notes hypotonia, irritability, and a poor sucking reflex in a full-term newborn on admission to the nursery. The nurse suspects fetal alcohol syndrome and is aware that which additional sign would be consistent with this syndrome? 1.Length of 19 inches 2.Abnormal palmar creases 3.Birth weight of 6 lb, 14 oz 4.Head circumference appropriate for gestational age

2.Abnormal palmar creases

A nurse is preparing a list of self-care instructions for a PP client who was diagnosed with mastitis. Select all instructions that would be included on the list. 1. Take the prescribed antibiotics until the soreness subsides. 2. Wear supportive bra 3. Avoid decompression of the breasts by breastfeeding or breast pump 4. Rest during the acute phase 5. Continue to breastfeed if the breasts are not too sore.

2, 4, and 5. Mastitis are an infection of the lactating breast. Client instructions include resting during the acute phase, maintaining a fluid intake of at least 3L/ day, and taking analgesics to relieve discomfort. Antibiotics may be prescribed and are taken until the complete prescribed course is finished. They are not stopped when the soreness subsides. Additional supportive measures include the use of moist heat or ice packs and wearing a supportive bra. Continued decompression of the breast by breastfeeding or pumping is important to empty the breast and prevent formation of an abscess.

A nurse is monitoring a new mother in the PP period for signs of hemorrhage. Which of the following signs, if noted in the mother, would be an early sign of excessive blood loss? 1. A temperature of 100.4*F 2. An increase in the pulse from 88 to 102 BPM 3. An increase in the respiratory rate from 18 to 22 breaths per minute 4. A blood pressure change from 130/88 to 124/80 mm Hg

2. During the 4th stage of labor, the maternal blood pressure, pulse, and respiration should be checked every 15 minutes during the first hour. A rising pulse is an early sign of excessive blood loss because the heart pumps faster to compensate for reduced blood volume. The blood pressure will fall as the blood volume diminishes, but a decreased blood pressure would not be the earliest sign of hemorrhage. A slight rise in temperature is normal. The respiratory rate is increased slightly.

The nurse is assessing a client who is 6 hours PP after delivering a full-term healthy infant. The client complains to the nurse of feelings of faintness and dizziness. Which of the following nursing actions would be most appropriate? 1.Obtain hemoglobin and hematocrit levels 2.Instruct the mother to request help when getting out of bed 3.Elevate the mother's legs 4.Inform the nursery room nurse to avoid bringing the newborn infant to the mother until the feelings of light-headedness and dizziness have subsided.

2. Orthostatic hypotension may be evident during the first 8 hours after birth. Feelings of faintness or dizziness are signs that should caution the nurse to be aware of the client's safety. The nurse should advise the mother to get help the first few times the mother gets out of bed. Obtaining an H/H requires a physicians order.

A nurse is teaching the mother of a newborn infant measures to maintain the infant's health. The nurse identifies which as an example of primary prevention activities for the infant? 1.Selective placement of the infant 2.Periodic well-baby examinations 3.Phenylketonuria (PKU) testing at birth 4.Administration of an antibiotic for an umbilical cord staphylococcal infection

2. Periodic well-baby examinations

The nurse is planning care for a newborn of a mother with diabetes mellitus. What is the priority nursing consideration for this newborn? 1.Developmental delays because of excessive size 2.Maintaining safety because of low blood glucose levels 3.Choking because of impaired suck and swallow reflexes 4.Elevated body temperature because of excess fat and glycogen

2.Maintaining safety because of low blood glucose levels

The nurse develops a plan of care for a woman with human immunodeficiency virus infection and her newborn. The nurse should include which intervention in the plan of care? 1.Monitoring the newborn's vital signs routinely 2.Maintaining standard precautions at all times while caring for the newborn 3.Initiating referral to evaluate for blindness, deafness, learning problems, or behavioral problems 4.Instructing the breast-feeding mother regarding the treatment of the nipples with nystatin ointment

2.Maintaining standard precautions at all times while caring for the newborn

The nurse in a newborn nursery is performing an assessment of an infant. What procedure should the nurse use to measure the infant's head circumference? 1.Wrap the tape measure around the infant's head, and measure just below the eyebrows. 2.Place the tape measure under the infant's head, wrap around the occiput, and measure just above the eyebrows. 3.Place the tape measure under the infant's head at the base of the skull, and wrap around to the front just below the eyes. 4.Place the tape measure at the back of the infant's head, wrap around across the ears, and measure across the infant's mouth.

2.Place the tape measure under the infant's head, wrap around the occiput, and measure just above the eyebrows.

Which of the following complications may be indicated by continuous seepage of blood from the vagina of a PP client, when palpation of the uterus reveals a firm uterus 1 cm below the umbilicus? 1. Retained placental fragments 2. Urinary tract infection 3. Cervical laceration 4. Uterine atony

3. Continuous seepage of blood may be due to cervical or vaginal lacerations if the uterus is firm and contracting. Retained placental fragments and uterine atony may cause subinvolution of the uterus, making it soft, boggy, and larger than expected. UTI won't cause vaginal bleeding, although hematuria may be present.

The nurse in the newborn nursery is preparing to complete an initial assessment on a newborn infant who was just admitted to the nursery. The nurse should place a warm blanket on the examining table to prevent heat loss in the infant caused by which method? 1.Radiation 2.Convection 3.Conduction 4.Evaporation

3.Conduction

The nurse is assessing a newborn after circumcision and notes that the circumcised area is red with a small amount of bloody drainage. Which nursing action is most appropriate? 1.Apply gentle pressure. 2.Reinforce the dressing. 3.Document the findings. 4.Contact the health care provider (HCP).

3.Document the findings.

The nurse assisted with the delivery of a newborn. Which nursing action is most effective in preventing heat loss by evaporation? 1.Warming the crib pad 2.Closing the doors to the room 3.Drying the infant with a warm blanket 4.Turning on the overhead radiant warmer

3.Drying the infant with a warm blanket

Which medication should the nurse plan to administer to a newborn by the intramuscular (IM) route? 1.Erythromycin 2.Tetracycline 1% 3.Phytonadione (Vitamin K) 4. Measles-mumps-rubella vaccination

3.Phytonadione (Vitamin K)

The nurse in the delivery room is performing an assessment on a newborn to determine the Apgar score. The nurse notes an Apgar score of 6. On the basis of this score, what should the nurse determine? 1.The newborn requires vigorous resuscitation. 2.The newborn is adjusting well to extrauterine life. 3.The newborn requires some resuscitative interventions. 4. The newborn is having some difficulty adjusting to extrauterine life

3.The newborn requires some resuscitative interventions.

A newborn is delivered via spontaneous vaginal delivery. On reception of the crying newborn, the nurse's priority is to perform which action? 1.Determine Apgar score. 2.Auscultate the heart rate. 3.Thoroughly dry the newborn. 4.Take the newborn's rectal temperature.

3.Thoroughly dry the newborn.

The nurse is monitoring a client who is receiving oxytocin (Pitocin) to induce labor. Which assessment finding would cause the nurse to immediately discontinue the oxytocin infusion? 1.Fatigue 2.Drowsiness 3.Uterine hyperstimulation 4.Early decelerations of the fetal heart rate

3.Uterine hyperstimulation

When caring for a client in the first stage of labor, the nurse documents cervical dilation of 9 cm and intense contractions lasting 45 to 60 seconds and occurring about every 2 minutes. Based on these findings, the nurse should recognize that the client is in which phase of labor? 1. Active phase 2. Latent phase 3. Descent phase 4. Transitional phase

4 RATIONALES: In the transitional phase, the cervix dilates from 8 to 10 cm, and intense contractions occur every 1½ to 2 minutes and last for 45 to 90 seconds. In the active phase, the cervix dilates from 5 to 7 cm, and moderate contractions progress to strong contractions that last 60 seconds. In the latent phase, the cervix dilates 3 to 4 cm, and contractions are short, irregular, and mild. No descent phase exists. (Fetal descent may begin several weeks before labor but usually doesn't occur until the second stage of labor.)

A nurse is developing a plan of care for a PP woman with a small vulvar hematoma. The nurse includes which specific intervention in the plan during the first 12 hours following the delivery of this client? 1.Assess vital signs every 4 hours 2.Inform health care provider of assessment findings 3.Measure fundal height every 4 hours 4.Prepare an ice pack for application to the area.

4. Application of ice will reduce swelling caused by hematoma formation in the vulvar area. The other options are not interventions that are specific to the plan of care for a client with a small vulvar hematoma.

Rho(D) immune globulin (RhoGAM) is prescribed for a client after delivery and the nurse provides information to the client about the purpose of the medication. The nurse determines that the woman understands the purpose if the woman states that it will protect her next baby from which condition? 1. Having Rh-positive blood 2. Developing a rubella infection 3. Developing physiological jaundice 4. Being affected by Rh incompatibility

4. Being affected by Rh incompatibility

The nurse is providing instructions to a new mother regarding cord care for a newborn infant. Which statement, if made by the mother, indicates a need for further instructions? 1. "The cord will fall off in 1 to 2 weeks." 2. "Alcohol may be used to clean the cord." 3. "I should cleanse the cord two or three times a day." 4. "I need to fold the diaper above the cord to prevent infection."

4. "I need to fold the diaper above the cord to prevent infection."

The nurse prepares to administer a vitamin K injection to a newborn, and the mother asks the nurse why her infant needs the injection. What best response should the nurse provide? 1. "Your newborn needs vitamin K to develop immunity." 2. "The vitamin K will protect your newborn from being jaundiced." 3. "Newborns have sterile bowels, and vitamin K promotes the growth of bacteria in the bowel." 4. "Newborns are deficient in vitamin K, and this injection prevents your newborn from bleeding."

4. "Newborns are deficient in vitamin K, and this injection prevents your newborn from bleeding."

A nurse employed in a neonatal intensive care nursery receives a telephone call from the delivery room and is told that a newborn with spina bifida (myelomeningocele type) will be transported to the nursery. The maternity nurse prepares for the arrival of the newborn and places which priority item at the newborn's bedside? 1. A rectal thermometer 2. A blood pressure cuff 3. A specific gravity urinometer 4. A bottle of sterile normal saline

4. A bottle of sterile normal saline

Which of the following responses is most appropriate for a mother with diabetes who wants to breastfeed her infant but is concerned about the effects of breastfeeding on her health? 1. Mothers with diabetes who breast-feed have a hard time controlling their insulin needs 2. Mothers with diabetes shouldn't breastfeed because of potential complications 3. Mothers with diabetes shouldn't breastfeed; insulin requirements are doubled. 4. Mothers with diabetes may breastfeed; insulin requirements may decrease from breastfeeding.

4. Breastfeeding has an antidiabetogenic effect. Insulin needs are decreased because carbohydrates are used in milk production. Breastfeeding mothers are at a higher risk of hypoglycemia in the first PP days after birth because the glucose levels are lower. Mothers with diabetes should be encouraged to breastfeed.

A nurse in a PP unit is instructing a mother regarding lochia and the amount of expected lochia drainage. The nurse instructs the mother that the normal amount of lochia may vary but should never exceed the need for: 1. One peripad per day 2. Two peripads per day 3. Three peripads per day 4. Eight peripads per day

4. The normal amount of lochia may vary with the individual but should never exceed 6 to 8 peripads per day. The average number of peripads is 6 per day

The nurse administers erythromycin ointment (0.5%) to the eyes of a newborn and the mother asks the nurse why this is performed. Which explanation is best for the nurse to provide about neonatal eye prophylaxis? 1.Protects the newborn's eyes from possible infections acquired while hospitalized. 2.Prevents cataracts in the newborn born to a woman who is susceptible to rubella. 3.Minimizes the spread of microorganisms to the newborn from invasive procedures during labor. 4.Prevents an infection called ophthalmia neonatorum from occurring after delivery in a newborn born to a woman with an untreated gonococcal infection.

4.Prevents an infection called ophthalmia neonatorum from occurring after delivery in a newborn born to a woman with an untreated gonococcal infection.

The nurse is preparing to administer an injection of vitamin K to a newborn. Which injection site should the nurse select? 1.The gluteal muscle 2.The lower aspect of the rectus femoris muscle 3.The medial aspect of the upper third of the vastus lateralis muscle 4.The lateral aspect of the middle third of the vastus lateralis muscle

4.The lateral aspect of the middle third of the vastus laterals muscle This muscle is the preferred injection site because it is free of major blood vessels and nerves and is large enough to absorb the medication.

The nurse is performing Apgar scoring for a newborn immediately after birth. The nurse notes that the heart rate is less than 100, respiratory effort is irregular, and muscle tone shows some extremity flexion. The newborn grimaces when suctioned with a bulb syringe, and the skin color indicates some cyanosis of the extremities. The nurse should most appropriately document which Apgar score for the newborn? 3 5 7 10

5

The nurse is checking a newborn's 1-minute Apgar score based on the following assessment. The heart rate is 160 beats/min; he has positive respiratory effort with a vigorous cry; his muscle tone is active and well-flexed; he has a strong gag reflex and cries with stimulus to the soles of his feet; his body is pink, with his hands and feet cyanotic. Which is the newborn's 1-minute Apgar score? 7 9 8 10

9

16. The nurse compared the birth weight of a 3-day-old with her current weight and determined the infant had lost weight. The most appropriate intervention by the nurse is: a. to do nothing because this is a normal occurrence. b. report the discrepancy to the pediatrician immediately. c. decrease the interval between the infant's feedings. d. try feeding the infant a different type of formula.

ANS: A It is typical for the newborn to lose 5% to 10% of his or her birth weight in the first 3 to 4 days of life. No change in the plan of care is needed.

4. When the newborn's crib was moved suddenly, the nurse noticed that his legs flexed and the arms fanned out, and then both came back toward the midline. The nurse would interpret this behavior as: a. the Moro reflex. b. the grasp reflex. c. an abnormality of the musculoskeletal system. d. a neurological abnormality.

ANS: A The Moro reflex is a normal neonatal reflex. It is elicited when the infant's crib is jarred. The infant responds by drawing the legs up, fanning the arms, and then bringing the arms to the midline in an embrace position.

21. The assessment of the newborn that should be reported is: a. head circumference that is 5 cm greater than the chest circumference. b. hands and feet that are cool and cyanotic. c. temperature of 36.2 C (97.1 F). d. mucus draining from nose.

ANS: A The circumference of the head should be less that 2 cm greater than that of the chest. All other listed assessments are within the norm.

12. The parents of a newborn girl express concern about the infant's vaginal discharge, which appears to be bloody mucus. The nurse explains that this is caused by: a. premature stimulation of the ovarian hormones by the pituitary system. b. cessation of female sex hormones transferred in utero from mother to infant. c. the increased amount of circulating blood from the mother throughout pregnancy. d. trauma to the genitalia during the birth process.

ANS: B Blood-tinged mucus discharged from the vagina is caused by hormonal withdrawal from the mother at birth.

8. The nurse is measuring the vital signs of a full-term newborn. An abnormal finding would be: a. an axillary temperature of 36.6° C (98° F). b. an apical pulse rate of 178 beats/min. c. respirations of 35 breaths/min. d. blood pressure of 80/50 mm Hg.

ANS: B The normal range for a newborn's pulse rate is 110-160 beats/min. A pulse rate outside of this range should be reported.

5. A first-time mother reports that she is experiencing difficulty breastfeeding her newborn. The neonatal reflex that the nurse would teach the mother to elicit, in order to facilitate breastfeeding, is: a. sucking. b. rooting. c. grasping. d. tonic neck.

ANS: B The rooting reflex causes the infant's head to turn in the direction of anything that touches the cheek in anticipation of food.

1. While inspecting a newborn's head, the nurse identifies a swelling of the scalp that does not cross the suture line. The nurse would document this finding as: a. molding. b. caput succedaneum. c. cephalohematoma. d. enlarged fontanelle.

ANS: C A cephalohematoma is caused by a collection of blood beneath the periosteum of the cranial bone. It does not cross the suture line.

20. To protect newborns from infection while in the nursery, the nurse plans to: a. keep the newborn dressed warmly. b. adjust room temperature between 23.8° C (75° F) and 26.6° C (80° F). c. wash hands before touching each infant. d. wear a disposable gown when giving infant care.

ANS: C Handwashing is the most reliable precaution available to prevent infection. The nurse washes his or her hands between handling different babies.

22. The nurse explains to an anxious parent that the dark areas over the sacrum of the newborn are a transitory skin discoloration called: a. Epstein's pearls. b. milia. c. stork bites. d. Mongolian spots.

ANS: D Bluish skin discoloration over the sacral area of a newborn is a transitory condition called Mongolian spots.

14. The statement that indicates the parents understand when to contact the pediatrician or nurse practitioner is that the: a. infant refuses a feeding. b. infant has an axillary temperature of 97° F. c. infant has three pasty, yellow-brown stools in 24 hours. d. infant's diaper is not wet after 8 hours.

ANS: D Decreased or lack of voiding by the newborn should be reported to the pediatrician or nurse practitioner to prevent dehydration.

3. Shortly after delivery, a symptom of respiratory distress in the newborn that should be reported is: a. cyanosis of the hands and feet. b. irregular heart rate. c. mucus draining from the nose. d. sternal or chest retractions.

ANS: D Sternal retractions are evidence that the newborn is in respiratory distress and should be reported immediately.

The nurse is caring for a woman receiving a lumbar epidural anesthetic block to control labor pain. What should the nurse do to prevent hypotension?

Answer: 4 RATIONALES: Because the woman is in a state of relative hypovolemia, administering fluids I.V. before the epidural anesthetic is given may prevent hypotension. Administration of an epidural anesthetic may lead to hypotension because blocking the sympathetic fibers in the epidural space reduces peripheral resistance. Ephedrine may be administered after an epidural block if a woman becomes hypotensive and shows evidence of cardiovascular decompensation. However, ephedrine isn't administered to prevent hypotension. Oxygen is administered to a woman who becomes hypotensive, but it won't prevent hypotension. Placing a pregnant woman in the supine position can contribute to hypotension because of uterine pressure on the great vessels.

A client with active genital herpes is admitted to the labor and delivery area during the first stage of labor. Which type of birth should the nurse anticipate for this client? 1. Mid forceps 2. Low forceps 3. Induction 4. Cesarean

Answer: 4 RATIONALES: For a client with active genital herpes, cesarean birth helps avoid infection transmission to the neonate, which would occur during a vaginal birth. Mid forceps and low forceps are types of vaginal births that could transmit the herpes infection to the neonate. Induction is used only during vaginal birth; therefore, it's inappropriate for this client.

The nurse is preparing to listen to the apical heart rate of a newborn. The nurse performs the procedure and should note that the heart rate is normal if which rate is noted? 1.A heart rate of 100 beats/min 2.A heart rate of 140 beats/min 3.A heart rate of 180 beats/min 4.A heart rate of 190 beats/min

.A heart rate of 140 beats/min

On completing a fundal assessment, the nurse notes the fundus is situated on the client's left abdomen. Which of the following actions is appropriate? 1. Ask the client to empty her bladder 2. Straight catheterize the client immediately 3. Call the client's health provider for direction 4. Straight catheterize the client for half of her uterine volume

1. A full bladder may displace the uterine fundus to the left or right side of the abdomen. Catheterization is unnecessary invasive if the woman can void on her own.

As part of the postpartum assessment, the nurse examines the breasts of a primiparous breastfeeding woman who is one day postpartum. An expected finding would be: 1. Soft, non-tender; colostrum is present 2. Leakage of milk at let down 3. Swollen, warm, and tender upon palpation 4. A few blisters and a bruise on each areola

1. Breasts are essentially unchanged for the first two to three days after birth. Colostrum is present and may leak from the nipples.

Which of the following factors might result in a decreased supply of breast milk in a PP mother? 1. Supplemental feedings with formula 2. Maternal diet high in vitamin C 3. An alcoholic drink 4. Frequent feedings

1. Routine formula supplementation may interfere with establishing an adequate milk volume because decreased stimulation to the mother's nipples affects hormonal levels and milk production.

A nurse performs an assessment on a client who is 4 hours PP. The nurse notes that the client has cool, clammy skin and is restless and excessively thirsty. The nurse prepares immediately to: 1. Assess for hypovolemia and notify the health care provider 2. Begin hourly pad counts and reassure the client 3. Begin fundal massage and start oxygen by mask 4. Elevate the head of the bed and assess vital signs

1. Symptoms of hypovolemia include cool, clammy, pale skin, sensations of anxiety or impending doom, restlessness, and thirst. When these symptoms are present, the nurse should further assess for hypovolemia and notify the health care provider.

Which of the following findings would be expected when assessing the postpartum client? 1. Fundus 1 cm above the umbilicus 1 hour postpartum 2. Fundus 1 cm above the umbilicus on postpartum day 3 3. Fundus palpable in the abdomen at 2 weeks postpartum 4. Fundus slightly to the right; 2 cm above umbilicus on postpartum day 2

1. Within the first 12 hours postpartum, the fundus usually is approximately 1 cm above the umbilicus. The fundus should be below the umbilicus by PP day 3. The fundus shouldn't be palpated in the abdomen after day 10.

The mother of a newborn calls the clinic and reports that when cleaning the umbilical cord, she noticed that the cord was moist and that discharge was present. What is the most appropriate nursing instruction for this mother? 1.Bring the infant to the clinic. 2.This is a normal occurrence. 3.Increase the number of times that the cord is cleaned per day. 4.Monitor the cord for another 24 to 48 hours and call the clinic if the discharge continues.

1.Bring the infant to the clinic.

The nurse is preparing to bathe a 1-day-old newborn. Which action should the nurse avoid when performing the procedure? 1.Immersing the newborn in water 2.Supporting the newborn's body during the bath 3.Ensuring that the water temperature is warm 4.Ensuring that the water temperature does not exceed 100° F

1.Immersing the newborn in water Newborn infants may be immersed in water after the umbilical stump has healed

The nurse is assessing the reflexes of a newborn infant. In eliciting the Moro reflex, the nurse should perform which action? 1.Make a loud, abrupt noise to startle the newborn. 2. Stimulate the ball of the foot of the newborn by firm pressure. 3. Stimulate the perioral cavity of the newborn infant with a finger. 4. Stimulate the pads of the newborn infant's hands by firm pressure.

1.Make a loud, abrupt noise to startle the newborn.

An initial assessment on a large-for-gestational age (LGA) newborn infant is being done. Which physical assessment technique should the nurse assist in performing to assess for evidence of birth trauma? 1.Palpate the clavicles for a fracture. 2.Auscultate the heart for a cardiac defect. 3.Blanch the skin for evidence of jaundice. 4.Perform Ortolani's maneuver for hip dislocation.

1.Palpate the clavicles for a fracture.

The nurse in a newborn nursery is monitoring a preterm newborn for respiratory distress syndrome. Which assessment findings would alert the nurse to the possibility of this syndrome? 1.Tachypnea and retractions 2.Acrocyanosis and grunting 3.Hypotension and bradycardia 4.Presence of a barrel chest and acrocyanosis

1.Tachypnea and retractions

The nurse examines a woman one hour after birth. The woman's fundus is boggy, midline, and 1 cm below the umbilicus. Her lochial flow is profuse, with two plum-sized clots. The nurse's initial action would be to: 1. Place her on a bedpan to empty her bladder 2. Massage her fundus 3. Call the physician 4. Administer Methergine 0.2 mg IM which has been ordered prn

2. A boggy or soft fundus indicates that uterine atony is present. This is confirmed by the profuse lochia and passage of clots. The first action would be to massage the fundus until firm, followed by 3 and 4, especially if the fundus does not become or remain firm with massage. There is no indication of a distended bladder since the fundus is midline and below the umbilicus

A postpartum nurse is preparing to care for a woman who has just delivered a healthy newborn infant. In the immediate postpartum period the nurse plans to take the woman's vital signs: 1. Every 30 minutes during the first hour and then every hour for the next two hours. 2. Every 15 minutes during the first hour and then every 30 minutes for the next two hours. 3. Every hour for the first 2 hours and then every 4 hours 4. Every 5 minutes for the first 30 minutes and then every hour for the next 4 hours.

2. Every 15 minutes during the first hour and then every 30 minutes for the next two hours.

Which of the following physiological responses is considered normal in the early postpartum period? 1. Urinary urgency and dysuria 2. Rapid diuresis 3. Decrease in blood pressure 4. Increase motility of the GI system

2. In the early PP period, there's an increase in the glomerular filtration rate and a drop in the progesterone levels, which result in rapid diuresis. There should be no urinary urgency, though a woman may feel anxious about voiding. There's a minimal change in blood pressure following childbirth, and a residual decrease in GI motility.

A newborn infant of a mother who has human immunodeficiency virus (HIV) infection is tested for the presence of HIV antibodies. An enzyme-linked immunosorbent assay (ELISA) is performed, and the results are positive. Which is the correct interpretation of these results? 1. Positive for HIV 2. Indicates the presence of maternal infection 3. Indicates that the newborn will develop AIDS later in life 4. Positive for acquired immunodeficiency syndrome (AIDS)

2. Indicates the presence of maternal infection

The nurse is assessing the lochia on a 1 day PP patient. The nurse notes that the lochia is red and has a foul-smelling odor. The nurse determines that this assessment finding is: 1.Normal 2.Indicates the presence of infection 3.Indicates the need for increasing oral fluids 4.Indicates the need for increasing ambulation

2. Lochia, the discharge present after birth, is red for the first 1 to 3 days and gradually decreases in amount. Normal lochia has a fleshy odor. Foul smelling or purulent lochia usually indicates infection, and these findings are not normal. Encouraging the woman to drink fluids or increase ambulation is not an accurate nursing intervention.

Methergine or pitocin is prescribed for a woman to treat PP hemorrhage. Before administration of these medications, the priority nursing assessment is to check the: 1. Amount of lochia 2. Blood pressure 3. Deep tendon reflexes 4. Uterine tone

2. Methergine and pitocin are agents that are used to prevent or control postpartum hemorrhage by contracting the uterus. They cause continuous uterine contractions and may elevate blood pressure. A priority nursing intervention is to check blood pressure. The physician should be notified if hypertension is present.

When performing a PP assessment on a client, the nurse notes the presence of clots in the lochia. The nurse examines the clots and notes that they are larger than 1 cm. Which of the following nursing actions is most appropriate? 1. Document the findings 2. Notify the physician 3. Reassess the client in 2 hours 4. Encourage increased intake of fluids.

2. Normally, one may find a few small clots in the first 1 to 2 days after birth from pooling of blood in the vagina. Clots larger than 1 cm are considered abnormal. The cause of these clots, such as uterine atony or retained placental fragments, needs to be determined and treated to prevent further blood loss. Although the findings would be documented, the most appropriate action is to notify the physician.

Which of the following changes best described the insulin needs of a client with type 1 diabetes who has just delivered an infant vaginally without complications? 1. Increase 2. Decrease 3. Remain the same as before pregnancy 4. Remain the same as during pregnancy

2. The placenta produces the hormone human placental lactogen, an insulin antagonist. After birth, the placenta, the major source of insulin resistance, is gone. Insulin needs decrease and women with type 1 diabetes may only need one-half to two-thirds of the prenatal insulin during the first few PP days.

The nurse has provided instructions about measures to clean the penis to a mother of a male newborn who is not circumcised. Which statement, if made by the mother, indicates an understanding of how to clean the newborn's penis? 1. "I should retract the foreskin and clean the penis every time I change the diaper." 2. "I need to retract the foreskin and clean the penis every time I give my infant a bath." 3. "I need to avoid pulling back the foreskin to clean the penis because this may cause adhesions." 4. "I should gently retract the foreskin as far as it will go on the penis and then pull the skin back over the penis after cleaning."

3. "I need to avoid pulling back the foreskin to clean the penis because this may cause adhesions." In male newborn infants, the prepuce is continuous with the epidermis of the glans and is not retractable. If retraction is forced, this may cause adhesions to develop. The mother should be told to allow separation to occur naturally, which usually occurs between 3 years and puberty. Most foreskins are retractable by 3 years of age and should be pushed back gently at this time for cleaning.

A nurse is caring for a PP client with a diagnosis of DVT who is receiving a continuous intravenous infusion of heparin sodium. Which of the following laboratory results will the nurse specifically review to determine if an effective and appropriate dose of the heparin is being delivered? 1. Prothrombin time 2. Internationalized normalized ratio 3. Activated partial thromboplastin time 4. Platelet count

3. Anticoagulation therapy may be used to prevent the extension of thrombus by delaying the clotting time of the blood. Activated partial thromboplastin time should be monitored, and a heparin dose should be adjusted to maintain a therapeutic level of 1.5 to 2.5 times the control. The prothrombin time and the INR are used to monitor coagulation time when warfarin (Coumadin) is used.

The nurse is caring for a post-term, small-for-gestational age (SGA) newborn infant immediately after admission to the nursery. What should the nurse monitor as the priority? 1. Urinary output 2. Total bilirubin levels 3. Blood glucose levels 4. Hemoglobin and hematocrit levels

3. Blood glucose levels

Which would be considered a normal finding in a newborn less than 12 hours old? 1.Grunting respirations 2.Heart rate of 190 beats/min 3.Bluish discoloration of the hands and feet 4.A yellow discoloration of the sclera and body

3. Bluish discoloration of the hands and feet

Which of the following findings would be a source of concern if noted during the assessment of a woman who is 12 hours postpartum? 1. Postural hypotension 2. Temperature of 100.4°F 3. Bradycardia — pulse rate of 55 BPM 4. Pain in left calf with dorsiflexion of left foot

4. Responses 1 and 3 are expected related to circulatory changes after birth. A temperature of 100.4°F in the first 24 hours is most likely indicative of dehydration which is easily corrected by increasing oral fluid intake. The findings in response 4 indicate a positive Homan sign and are suggestive of thrombophlebitis and should be investigated further.

During assessment for admission to the labor and delivery area, a client and her husband ask the nurse whether their sons, ages 8 and 10, can witness the childbirth. Before answering this question, the nurse should consider which guideline? 1. The children and client should share a support person during the childbirth. 2. Children should attend childbirth only if it takes place at home. 3. Children shouldn't attend childbirth because it will frighten them. 4. Each child attending the childbirth should have a separate support person.

Answer: 4 RATIONALES: Each child attending the childbirth should have a support person — one who isn't also serving as the client's support person. The support person explains what is happening, reassures the child, and removes the child from the area if an emergency occurs or if the child becomes frightened. Children can attend childbirth in any setting. The decision to have a child present hinges on the child's developmental level, ability to understand the experience, and amount of preparation.

For a client who's fully dilated, which of the following actions would be inappropriate during the second stage of labor? 1. Positioning the mother for effective pushing 2. Preparing for delivery of the baby 3. Assessing vital signs every 15 minutes 4. Assessing for rupture of membranes

Answer: 4 RATIONALES: In most cases, the membranes have ruptured (spontaneously or artificially) by this stage of labor. Positioning for effective pushing, preparing for delivery, and assessing vital signs every 15 minutes are appropriate actions at this time.


Kaugnay na mga set ng pag-aaral

Constitutional Law I: Federalist Papers

View Set

Independent Samples & Paired Samples t-tests

View Set

Pathophysiology: Chapter 33- Disorders of Thermoregulation

View Set